Skip to Content

NEW JERSEY BOARD of BAR EXAMINERS

NEW JERSEY BOARD of BAR EXAMINERS

Independence - Integrity - Fairness - Quality Service

Bar Examination Sample Q&A - February 2007

 

Questions and Sample Answers

February 2007

Civil Procedure
Criminal
Constitutional Law
Property
Evidence
Contracts
Torts
 

Question #1
Torts

Barry hosted a barbecue at his home.  In preparation, he purchased charcoal and lighter fluid, storing those items in his garage.

On the day of the party, Barry asked his 10-year-old son, Tony, to bring those items from the garage to the backyard, where Barry loaded the charcoal onto his grill, doused the charcoal, and then set fire to it. Meanwhile, Barry’s guests had arrived, and many congregated near the grill.

Immediately after Barry lit the charcoal, an explosion occurred, badly injuring a guest, Cynthia. Another guest, Phil, volunteered to driver Cynthia to the local hospital, which had a renowned burn center. The drive would normally take 10 minutes. Unfortunately, Phil and Cynthia became stuck in traffic backed up because of an accident. As a result, it took Phil an hour to reach the hospital.

Months later, Cynthia comes to your law office, seeking advice as to whether she has a personal injury claim. You begin an investigation and reach the following conclusions:

(A)    Barry’s son, Tony, mistakenly gave his father a can of kerosene, rather than lighter fluid. The two cans are of similar size and shape. The kerosene exploded when Barry lit the charcoal.

(B)     The kerosene, manufactured and packaged by ABC Co., was stored in a can that contained a label, warning that it was highly flammable and potentially explosive. However, Barry had purchased the kerosene five years earlier and, due to the passage of time, the warning label had faded and was no longer readable.

(C)     The accident that delayed Phil and Cynthia en route to the hospital was caused solely by Ron, who was driving under the influence. The wreckage of Ron’s car caused the blockage. Because of the extra time it took to reach the hospital, Cynthia’s recovery was substantially extended, and her out-of-pocket medical costs were doubled.

(D)    As it turned out, Phil could have avoided the traffic blockage by taking a shortcut to the hospital, a route he knew well. However, due to the stress of the incident, he did not think of the shortcut until after he arrived at the hospital.

The senior partner asks you to prepare a memorandum advising Cynthia of the claims she could assert against each potential defendant and the likely defenses each would raise.

Prepare the memorandum

 

Torts
Sample Answer #1

TO: SENIOR PARTNER

Re:  Claims of Cynthia

You have asked me to prepare a memorandum advising our client, Cynthia, of all claims she could assert against each defendant in the facts that she provided, and each defendant’s potential defenses.  A detailed discussion follows. 

NEGLIGENCE

Cynthia’s claims appear to sound in tort, and to give to raise to potential tort liability via a vis a host of potential defendants.  Most of these defendants will be subject to claims arising from a negligence standard.  Under this doctrine, a person is require to act in a manner consistent with the actions of a reasonably prudent person (i.e., a person is required to exercise the ordinary care of a person in his of her circumstances).   Negligence includes five elements.  First, a person must be shown to have a duty to act with reasonable care toward all forseeable persons who may be injured by his or her conduct.  Second, a breach of that duty must be established; in other words, the person must be shown not to have acted reasonably with regard to those persons owed a duty.  Third, the person’s breach of the duty must be the actual cause, or cause-in-fact, of the injured person’s harm.  This is generally resolved according to a “but for” test - - but for the actions of the potential defendant, breaching a duty, would the injured party have suffered the harm that he or she did?  Fourth, the person’s breach of the duty must also be the legal, or proximate cause of the harm suffered by the injured party.  Legal/proximate cause is a legal fiction that takes into account whether or not the injured party was a foreseeable plaintiff, and whether or not the harm suffered was too remote in time or nature for liability to attach to the presumptively negligent actor.  Finally, an injured party must show damages in order to recover. 

1)         Cynthia v. Barry

Cynthia may have a negligence claim against Barry.  On the facts presented, Barry owed a duty of reasonable care to Cynthia, who, as a social guest in Barry’s home, was a licensee.  A licensee must be warned of all known dangers that exist on the property of the person extending the license.  A question of fact may arise as to whether or not Barry was acting reasonably in loading charcoal onto his grill and dousing it with lighter fluid, then igniting it.  This analysis will comprise an assessment of whether or not Barry was acting reasonably in using combustible materials in close proximity to his guests, and whether or not he used reasonable care in the process, given the dangerous nature of the activity.  Finally, any trier of fact of Cynthia’s claims will examine whether or not Barry could have reasonably expected that an explosion would occur when he ignited the kerosene.  Finally, Barry will be held to the same standard of reasonable care with regard to his use of what he thought was the lighter fluid - - a jury question may arise as to whether or not Barry should have inspected the can of kerosene and discovered that it was not lighter fluid, which might have avoided the accident altogether.  With regard to all of the above, if Barry did not act reasonably, he will be found to have breached his duty of care owed to Cynthia as a social guest/licensee in his home. 

Actual cause will not be a problem here, as the explosion that Barry directly caused Cynthia’s harm.  Legal cause is a closer question, because Barry could raise the defense that he could not have foreseen that using what he thought was lighter fluid, in connection with the purpose for which it was intended (use of a grill in the backyard, outdoors), would cause an explosion.  It is possible that the explosion might be viewed as a supervening cause, which Barry could not have foreseen, cutting off his liability to Cynthia.  Another potential supervening cause might be the fact that Barry received the can of kerosene from his son, Tony, and could have not have foreseen However, since Cynthia was standing near the grill, she should be a “foreseeable plaintiff” under the general judge-made analysis of legal cause.  Finally, it does appear that Cynthia suffered damages. 

As defenses, Barry will no doubt raise the supervening causes referred to above, including the fact that he thought the can contained lighter fluid, but was misled by Tony into thinking it was lighter fluid, instead of a substance that was far more dangerous.  However, a jury will consider whether or not it was reasonable for Barry to enlist the participation of a 10-year old in bringing him combustible chemicals; this may be found to be a “nondelegable duty” or an adult activity, which it was not reasonable for Barry to delegate to Tony. 

Barry will also raise as a defense that he could not have foreseen that the lighter fluid (as he thought it to be) will explode, as mentioned above.  This analysis will likely turn on whether or not it was reasonable for Barry to inspect the can, which, although similar in dimensions to the lighter fluid can, was apparently labeled as kerosene - - even if the label was no longer readable.  Barry could have tested the substance to determine what it was before using it. 

Barry may also raise the defenses of contributory and/or comparative negligence and assumption of the risk against Cynthia.  Contributory negligence by a plaintiff willl operate as a complete bar and cut off the plaintiff’s claims.  Comparative negligence will either (depending on the laws of the jurisdiction involved)  reduce the plaintiff’s damages in proportion to the level of fault she suffered (pure comparative negligence), or, in a modified comparative negligence system, will cut off the plaintiff’s claims entirely if her negligence exceeded that of the wrongdoer.  Here, it does not seem likely that Cynthia was negligent, although a jury will examine whether or not she was standing too close to the grill when the explosion occurred.  Assumption of the risk is a similar doctrine that operates to limit or eliminate a plaintiff’s recovery when the person acts in reckless disregard of a known risk.  This does not seem to be a problem for Cynthia, since she probably reasonably did not foresee that, by congregating near Barry’s grill, she was subjecting herself to the risk of an explosion. 

Barry may also be sued for battery by Cynthia - - battery is an intentional harmful or offensive touching with the person of the plaintiff, which was not consented to.  The contact need not be direct, so Cynthia could recover, but Barry does  not seem to have acted intentionally, which will defeat liability under that theory here. 

Finally, Barry may be vicariously liable for the actions of Tony, his minor child, meaning that he will be responsible for Tony’s actions, and the fault for those actions will be imputed to Barry.  Normally, parents are not so liable unless they have knowledge of their children’s propensities to commit intentional torts.  Here, Tony does not appear to have done anything intentional.  However, vicarious liability may still arise if a jury finds that it was unreasonable for Barry to delegate the duty of handling combustible chemicals to his 10-year old child. 

2)       Cynthia v. Tony

Cynthia may sue Tony, but a child at the age of 10 will usually be held to a lower standard of duty regarding negligence, because of the child’s inexperience in appreciating known risks.  In addition, it does not appear from the facts that Tony acted unreasonably - - he did what his father asked him to.  A trier of Cynthia’s claims may not find that a 10-year old child would know the difference between kerosene and lighter fluid, and thus, Tony may have acted reasonably and not breached his duty of care.  Note that on occasion, children engaged in “adult activities” will be held to an adult standard, in which case Tony would have all of the same potential exposure to liability and defenses as his father, Barry, discussed above.   On balance, however, Cynthia’s claim against Barry (who assuredly has more financial wherewithal than his son) is probably stronger. 

3)         Cynthia v. Phil

Cynthia may proceed against Phil on a negligence theory.  Here, the question will be whether Phil acted reasonably in his rescue efforts.  A preliminary question might be whether it was more reasonable to call for professional medical assistance for Cynthia, such as an ambulance, rather than volunteering to drive her to the hospital.  This will be relevant because an ambulance, empowered with the legal ability to circumvent traffic, would not have encountered the difficulties that Phil did.  (Incidentally, Barry may be liable on this theory too, if it was not reasonable for him to fail to suggest that he or someone else call 911 and arrange for medical transport of Cynthia). 

Although a person generally has no duty to act to eliminate harm that they have not caused, once a person undertakes a rescue, as Phil did, they must proceed in a reasonable manner.  Phil will be held to the standard of a reasonable person of like experience and knowledge under the circumstances.  The facts indicate that he could have taken a shorter route to the hospital, but did not because of “stress” - - a jury will determine whether or not Phil’s failure to use a well-known shortcut that he was aware of was reasonable conduct in the process of his rescue efforts.  Phil may also be held negligent for the very fact that traffic delayed Cynthia’s arrival at the hospital.  As mentioned above, it is not clear why Phil did not call 911, and drove Cynthia himself.  Traffic is certainly a foreseeable consequence, and thus under the requisite causation analysis, Phil’s undertaking of a rescue that might have been accomplished more quickly by professionals, and being delayed by traffic, might certainly be a legal cause  of Cynthia’s  injuries.  Phil’s conduct is certainly a cause-in-fact of Cynthia’s injuries, and Cynthia did suffer damages. 

Phil will raise the defense that Ron’s driving while intoxicated, and the resulting traffic, were supervening causes operating to cut off his liabillity.  However, “danger invites rescue,” and traffic (from whatever source) is probably a foreseeable consequence whenever anyone takes to the road, let along carrying an injured person in need of critical medical care.   

4)         Cynthia v. Ron

Cynthia may bring a claim against Ron, who caused the traffic that delayed her arrival at the hospital.  Ron had a duty to act reaosnably, and he almost certainly breached it by driving while intoxicated in reckless disregard of a known risk (and the fact that he apparently did so in the middle of the day will probably not engender him any sympathy).  He may have owed a duty to Cynthia, as well as anyone else who might have been injured by his conduct - -  but here, a court might well use legal cause to cut off Ron’s liability, since it might not be foreseeable that his conduct, at the time he undertook it, would cause a traffic jam that would delay an injured woman being transported (for some reason) by a civilian volunteer form reaching the hospital.  Again, Ron will not be a sympathetic defendant, but he may ask the judge for a directed verdict or summary judgment declaring that he was not legally responsible due to legal cause, cutting off his liability and preventing the jury from having an opportunity to punish him.

Ron might also be liable under the doctrine of “negligence per se,” in which the standard of care is borrowed from the criminal law if a defendant violates a criminal statute.  Ron evidently did so, and the inquiry then would be whether or not Cynthia was in the class of persons sought to be protected by the statute, and whether or not her harm was within the class of risk contemplated by the statute.  Presumably, the legislature did not have people like Cynthia suffering extremely indirect injuries in mind in enacting the statute prohibiting or criminalizing drunk driving, but this argument is worth making, because if it is accepted by the court, Ron will be automatically negligent, and he and/or his insurer will be liable to Cynthia for her injuries. 

5)                  Cynthia v. ABC Co.

Cynthia may proceed against ABC, a commercial supplier of products, under a products liability negligence theory.  However, this may be a problematic claim, since ABC appears to have acted reasonably in placing a warning label on its product.  A higher standard may apply if the court deems the kerosene to be inherently dangerous, however, so this argument is worth making.  ABC’s kerosene was the cause in fact of Cynthia’s injuries, but it may not have been the legal cause, if Barry’s use of the product in a different manner than its intended use was a supervening cause, because this will cut off ABC’s liability.  ABC will doubtless raise this defense against Cynthia.   

ABC may also be strictly liable as a commercial supplier of products, which does not require proof of the elements of a negligence claim.  Strict products liability can arise from a manufacturing defect (the product is different from all others of its kind due to some defect), a design defect (the product is designed defectively), or a failure to warn.  No manufacturing defect seems present here, and although kerosene may have been designed differently, it is likely that the cost to make it safer might outweigh its utility, preventing liability from attaching under that theory.  Failure to warn is probably the strongest argument here, because ABC has a duty to warn users of all foreseeable risks.  However, the warning label likely accomplished this, assuming that ABC bears no fault in designing a warning label that faded over a five-year period.  ABC will also raise the defense that no one should have been using kerosene in the manner that Barry mistakenly did, so the misuse was not forseeable and ABC should not be liable. 

In any case, Cynthia will probably be found to be within the “zone of danger” of the use of ABC’s product - - it is not necessary that she have purchased it.  Mere bystanders can recover in strict liability.   However, assuming that ABC can establish the above defenses, this is not Cynthia’s strongest claim, but it may be worth making against a presumed “deep pocket” defendant such as ABC.   

6)         Cynthia v. Other Guests

Cynthia appears to have no claim against the other guests at Barry’s party, who did not participate in causing her injuries in any way - - unless they should have called 911 when they learned of the injury, or prevented Phil from undertaking his gratuitous rescue efforts.  However, it does not appear that any of them owed a duty to Cynthia, as no duty is implied at common law in such situations, and they did not apparently cause Cynthia’s injuries in any way. 

Finally, please note that all defendants may be expected to raise the defenses of contribution, seeking to reduce their own fault and allocate it to other tortfeasors.  Generally, a court will use joint and several liability where several tortfeasors combine to cause one indivisible set of injuries, and the burden will shift to the defendants to allocate fault among themselves.  In this case, a court might also find that several factors combined to cause Cynthia’s injuries, and apportion the fault on its own initiative.  Note that ABC Co., to the extent that it is strictly liable, may not be involved in such an allocation of fault, but from a negligence perspective, it certainly would be.     

 

Torts
Sample Answer #2

To:       Cynthia

From:   Associate

Re:       Your Potential Personal Injury Claims

I.          Cynthia v. Barry

First, you could assert a negligence claim against Barry. To establish a negligence claim, a plaintiff must prove that: (1) the defendant owed a duty of care to the person; (2) the defendant breached the duty of care; (3) the breach both factually and legally caused the plaintiff’s injury; and (4) the plaintiff suffered damages.  Parties always owe a duty to forseeable victims of their negligence, which is clearly applicable here because you were an invited guest to Barry’s home and therefore it was forseeable that you could suffer injuries that.  Normally, the duty owed is based on how a reasonably prudent person would have acted under the circumstances.  Some defendants owe a special duty of care based on their status, including occupiers of land, whose duty to licensees (people that come onto the land for their own purposes but with with the land occupier’s position, i.e. social guests) requires that they warn or fix dangerous conditions on the land that are hidden to the licensee and known to the occupier.  However, because your claim is with regard to activities that took place on the land, Barry owed you the duty of a reasonably prudent person who was barbecuing. 

The issue is whether Barry breached this duty by asking Tony, his 10 year old son, to go retreive the lighter fluid.  You could argue that reasonably prudent people would go get the flammable material they planned to ignite the grill with themselves, rather than sending their children to do so.  This argument is persuasive because Tony was 10 years old –the age of a third or fourth grader in elementary school – and although he could probably read at his age, no reasonably prudent person would send a child alone to get flammable fluid.  Although it was the day of he party, the facts do not indicate that the circumstances were exigent enough to warrant entrusting a child to do so.  Furthermore, Barry did not act reasonably because he did not double check the bottle of fluid that Tony brought back for him before dousing the charcoal with it.  After sending a 10 year old to get the bottle, the least he could have done was read its label to ensure that Tony brought back the right one.  Thus, Barry breached his duty of care to you by entrusting Tony to retrieve the liquid and not checking the bottle. 

Under these facts, it is clear that this breach both factually caused (“causation in fact”) and legally caused (“proximate causation”) you injuries.  Causation in fact exists when a plaintiff would not have been injured but for the defendants actions; proximate causation exists when the plaintiff’s injuries were forseeable based on the defendant’s breach.  But for Barry’s breach, you would not have been injured by the explosion.  Likewise, you were also a forseeable victim of this breach since you were an invited guest and Barry should have known that his breach could have caused injury to you or anyone else at the party.  Finally, you are able to prove damages.  Based on my investigation, you have incurred substantial out of pocket medial expenses as a result of this injury and subsequent events.

II.        Cynthia v. Tony

It is possible for you to pursue a negligence claim against Tony, however it will probably not be unsuccessful.  When children are between the ages of 4 and 18 years old, they are not held to the same standards as adults.  Their duty of care is defined by how a reasonable child of similar age, experience, and education would have acted under the circumstances.   This is a very broad standard and is usually very favorable for children.  Tony did not breach this standard because a reasonable 10 year old child would have probably made the same mistake Tony did and choose the wrong bottle due to unfamiliar words used on the label; the fact that the label on this bottle was faded; and the similar size and shape of the bottle.  Thus, because it is unlikely that you can prove Tony did breach this standard, this action would probably fail. 

It should also be noted that even if you could prove Tony’s negligence, it would be very difficult to collect a judgment from him and you would not be able to hold Barry responsible under a theory of vicarious liability.  A party may be held vicarious liable for the torts of others based on their relationship with that party (e.g. when employers are held liable for acts of their employees).  As a general rule, parents are not held liable for the torts of their children. Thus, you would not be able to collect against Tony either.

III.       Cynthia v. ABC Manufacturing Co.

You may also try to assert a products liablity claim against ABC for your injuries because they manufacture the kerosene.  This can be done as a strict liability claim or a negligence claim. 

A.  Strict Liability

Strict liability torts always impose liaiblity on defendants when they engage in dangerous activities or sell defective products (without regard to any safety measures the defendants have attempted).  A strict liability claim for defective products will allow a plaintiff to sue anyone in the chain of distribution (from the merchant who sold the product to the original manufacturer) if the plaintiff can prove that: (1) the party was a part of the distribution claim; (2) the product is defective (prove that the product is unreasonably unsafe because of a manufacturer defect or a design defect); (3) the product was defective when it left the manufacturer’s hands; and (4) it was a forseeable user making forseeable use of this product.  This will be a difficult claim to prove.  ABC will probably argue that, although it is a part of the distribution chain, elements 2, 3, and 4 are not met under these facts.  A product can be defective by design because it lacks an adequate warnings or there was a hypothetical alternative design that was economically feasible, available at the time of production, and was not cumbersome.  It is highly unlikely that the kerosene is defective for its intended use as a manufacturer defect.  However, in terms of a design defect, you may have a claim.  ABC will probably argue that there was no design defect because this bottle did contain a label which warned that it was highly flammble and potentially explosive.  However, this label was ineffective because it had worn off after only 5 years; thus, there was a design defect because there was not adequate warning.  However, this claim will probably not be successful because element 3 is not met since Barry was not making forseeable use of the product.

B.  Negligence

You have a much better chance of raising the products liability claim as one of negligence.  ABC had a duty to act as the reasonable maker/seller of kerosene, which includes giving adequate warning.  A reasonable purveyor of kerosene includes a warning that is not only highly descriptive, conspicuous, and clear, but also one that remains legible on a product for a reasonable amoung of time.  While it might be expected for a label to wear off at some point, a reasonable seller of something as dangerous as kerosene would ensure the label lasted at least 5 yeras.  Thus, ABC breached its duty of care because the label was inadequate since it had faded over the years and was no longer readable.  However, the factual and legal causation will be difficult to prove, although it is arguable that but for the lack of a legible label, Barry or Tony would have noticed the error and that this lack of a label could forseeably result in a person being burned. 

IV.       Cynthia v. Phil

You also have a cause of action against Phil.  There is no affirmative duty to rescue others (unless this duty is imposed by statute or because some special relationship between the parties exists).  However, when one undertakes a rescue, he or she must rescue with reasonable care.  In other words, a party may be liable for his own negligence during a rescue.  In this case, Phil undertook the duty to rescue you and was required to do so reasonably.  It appears as though he did not.  It was not reasonable under these circumstances to sit in traffic for an hour for a trip that should have taken 10 minutes, especially when Phil knew of a shortcut.  When parties have a superior knowledge (even just a nugget of information), they are required to use that information.  Because Phil knew of this shortcut, he should have employed it earlier in the hour.

Phil will probably try to defend on the grounds that he was could not think clearly because of the stressful situation.  However, the facts indicate that he was thinking very clearly because he recommended a good hospital burn unit and attempted to drive you there.  Furthermore, a reasonable rescuer would not get so stressed over these facts so as to allow a traffic jam to lengthen your trip to the hospital.  Thus, Phil breached his duty to you.  Causation is met because but for his inaction, your injuries would not be as severe, which was also a forseeable result of his breach. Finally, you have asserted that your damages were increased becasue it took so long to get to the hospital.

V.        Cynthia v. Ron

However, you will not have such a claim against Ron because Ron owed no duty of care to you.  Under the Palsgraff approach, people only owe duties of care to forseeable victims of their negligence.  You were not a forseeable victim because there is no way that he could expected that his negligence in drunk driving would cause a burn victim’s injuries to be exacerbated.  (Should you choose to file in one of the rare jurisdictions that has adopted the Andrews approach – which holds that people owe a duty of care to the whole world – we can revisit this point.)

Likewise, a negligence per se argument will also be unsuccessful.  To borrow a duty of care from a criminal statute, the plaintiff must prove he or she was part of the class of persons to be protected and suffered injury from the class of risks the statute sought to prevent.  Assuming that there is such a criminal statute (which is almost certain because my research shows no state that does not criminalize drunk driving), plaintiff does not meet either element.  Such a statute seeks to protect other drivers from injury caused by a drunk driver, not to protect drivers from being delayed because of drunk drivers whether or not these people are rushing to the hospital.  Further, your injury is not within the class of risks the statute seeks to prevent: injuries caused by drunk drivers, not injuries worsened by drunk drivers.  Thus, there is no negligence claim against Ron because he owed you no duty.

VI.       Additional Defenses

There are a few additional arguments that the defendants Barry, ABC, and Phil can raise as defenses: 

(1) Limited Damages for Subsequent Parties’ Negligence:  Barry may argue that he should not be held liable for the extended damages you suffered because of the elongated trip to the hospital.  He may argue that his damages should be limited because those extra costs should not be attributed to him because Phil’s negligence was an superceding cause that broke the chain of causation, thus releasing Barry from any additional damages you suffered as a result of the drive to the hospital taking an hour when it should have only taken 10 minutes. 

This argument will not be successful.  When parties act negligently, they are held liable for all forseeable subsequent injuries and accidents from other parties’ negligence, including neligent medical care and negligent rescue, because these results are forseeable.  As Justice Cardozo once said, “danger invites rescue.”  In this case, Barry will be liable for the injuries you suffered from the explosion because any alleged negligence of Phil (discussed below) as a rescuer would have been forseeable, and therefore not break the chain of causation.

(2) Assumption of the Risk.  All defendants may also argue that you assumed the risk of injury by “congregating near the grill” when he set fire to it.  Assumption of the risk applies when a party is aware of the risks attendant to the behavior and then consents to those risks, and is usually only applicable to minor injuries (e.g. parties horseplaying assume the risk that their activities may result in a few scratches, etc). First, it is not clear that you were one of the guests that went close to the fire; but even assuming that you were, this defense is not applicable to you.  First, the risks attendant to standing near a grill when someone is using lighter fluid are obviously lower than those when someone is using kerosene, so you were not aware.  Furthermore, your conduct does not indicate any type of consent to severe injuries from an explosion. 

(3) Contributory Negligence.  The defendants will also probably raise the same arguments as those re assumption of the risk and assert that you were contributorily negligent.  A party may be held contributorily when he or she does not act as a reasonable person with due care for his or her own safety.  Contributory negligence precludes that party’s ability to recover any damages.  This defense will probably not be successful because the facts do not indicate that you were unreasonable in acting for your own safety.  A reasonably prudent person at a barbeque often goes near the grill without fear of explosions (i.e. to retrieve more food, talk to the cook), as did many of the party goers at Barry’s house.  Because you acted like the reasonable person, you were not contributorily negligent.  The applicability of this rule will also be determined based on where you file suit, as all states do not apply this rule.  New Jersey applies comparative negligence, as discussed below.

(4) Comparative Negligence.  Unlike contributory negligence, comparative negligence allows the jury to determine the level of liability attributable to each person (usually done in terms of percentages).  If the plaintiff is liable at all, the judge then reduces his or her recovery by the plaintiff’s allocation of fault.  All defendants may raise the same arguments as those re assumption of the risk and contributory negligence to assert that any of your recovery should at least be reduced by your own liability.  Because I do not believe that these facts indicate your liability, this will probably not apply.  However, you should expect that the tortfeasors will be jointly and severally liable to each other and seek contribution from any other potential tortfeasor (e.g., Barry will attempt to seek contribution from Phil because he negligently transported you of from ABC if you can establish their negligence).

I hope this was helpful and look forward to working with you.

 

Question #2
Contracts

In 1999, in connection with John’s promotion to Chief Financial Officer of publicly-traded Company, John entered into a written indemnification agreement (“Agreement”) with Company whereby Company agreed to indemnify John for any “investigations, claims, actions, lawsuits, etc.” and “any costs associated therewith” arising out of his employment with Company. The Agreement also provided Company would pay any costs associated with John’s enforcement of the Agreement, including legal fess and expenses. Further, “Company shall pay all such costs forthwith upon receipt.”

In 2001, Company entered into a written Insurance Contract with Insurer that covered any claims filed against the Company or its officers. The Insurance Contract further required that Company and its officers choose an attorney from Insurer’s approved list (“Panel Counsel”) to represent them.

In 2003, John left Company, but before he did, he and Company entered into a written Separation Agreement. The Separation Agreement paid John $850,000 severance pay and provided John must “cooperate” with the Company if any “claims, actions, lawsuits, etc.” are brought against Company. Under the Separation Agreement, John’s failure to “cooperate” could be deemed a breach of contract and could result in forfeiture of the severance pay.

In 2004, Company restated its earnings for the years 2000 through 2002, which spawned an SEC investigation, an internal Company investigation, and eight class-action lawsuits (all of which accused Company, John, and several other current and former corporate employees of improper financial dealings such as insider trading). Company’s in-house General Counsel/Corporate Secretary told John to hire an attorney, later in writing approved the attorney John hired, and promised Company would indemnify John as required by the Agreement. John’s attorney (“Attorney”) undertook the defense of John with respect to all those matters and submitted “undetailed” legal bills to Company.

Once Company received the legal bills, both Company’s In-house General Counsel and Outside Counsel contacted John directly and told John they wanted to discuss “legal strategy.” General Counsel also told John that Company needed detailed legal bills setting forth the steps taken and defenses being considered on John’s behalf in order to evaluate whether the bills were “reasonable.” To protect John, his Attorney refused to submit “detailed” legal bills. The Company refused to pay John’s $255,000 legal fees, revoked approval of John’s Attorney, and told John he must choose Panel Counsel to represent him per the Insurance Contract. Company also opined John was in violation of the Separation Agreement.

John does not want Panel Counsel but wants his legal bills paid so his Attorney can continue defending all matters. John sues the Company, and Company files a countersuit. You are the law clerk to the trial judge who has asked you to prepare a memorandum setting forth and evaluating all possible causes of action, defenses, and damages claims John or Company may present. The Court also requests you advise her of any improper attorney conduct.

Prepare the Memorandum

Contracts
Sample Answer #2A

Memorandum

To: Trial Judge

From: Clerk

Re: John and Company’s claims and counterclaims/possible attorney misconduct

I. John’s causes of action against Company

John will most likely prevail in a cause of action against Company for breach of contract. The issue is whether Company’s performance under the contract was excused or whether they breach the contract/indemnification agreement.

The Company and John had a valid contract/indemnification agreement. Therefore, the Company must adhere to the terms of the contract or they will be in breach, unless they had a valid excuse for their non-performance. This was a contract under common law and so the courts will look to the contract to see what the conditions of the performance are.

In this case, Company agreed to indemnify for “any investigation, claims, actions, lawsuits” and “any costs therewith” upon receipt of such costs, arising out of his employment with the Company, including any costs associated with enforcement of the indemnification agreement. The Company breached the indemnification agreement by refusing to pay for John’s legal bills unless they were “reasonable” and “detailed,” there are not such qualifiers in the Indemnification Agreement. Clearly the SEC investigation, internal investigation and class action lawsuits arose out of John’s employment with the Company during the time when he was employed there.

The Company will most likely defend the breach of contract claim by claiming that their performance under the Indemnification Agreement was excused by a later contract, the Insurance Contract. Therefore, they are not required to perform under the Indemnification Agreement with respect to John’s legal bills because the Insurance Contract replaced or substituted or modified Indemnification agreement and John must use Panel Counsel. The company may claim that John agreed to this because he knew about the Insurance Contract because it covered all officers of the Company and was entered into when John was an officer in 2001.

However, this defense of the Company’s will most likely fail because while John may have been a third-party beneficiary of the Insurance Contract, he did not expressly agree to substitute it for the Indemnification Agreement. The Insurance Contract was not an accord and satisfaction for the Indemnification Agreement as the Company may claim. Also, the General Counsel agreed in writing to indemnify John under that agreement in 2004, so the Company may not claim the Insurance Contract substituted performance of the Indemnification Agreement.

John is entitled to expectation damages, to put him in the same position as if the contract had been performed. Also, he is entitled to all costs of bringing the action against Company, including possibly consequential damages if he can prove he suffered any special damages the Company knew about when he entered into the indemnification Agreement.

II. Company’s counterclaim against John

Company will not prevail against John for a breach of the Separation Agreement. The issue is whether John’s actions in not submitting “detailed legal bills” amounts to his “failure to cooperate.”

Company will claim that John had a duty under the contract to cooperate if any claims were brought against the company, and that by insisting on his own counsel and failing to provide legal bills that were detailed, or by failing to discuss legal strategy, he breached the Separation Agreement.

However, John can adequately defend such a claim. First, he did cooperate by hiring an attorney, as advised and guided by the Company’s in-house counsel. Because John is represented by outside counsel, what he and the counsel discuss are Attorney-Client Privileged, and therefore Company is not entitled to those communications to discuss their legal strategy or entitled to detailed legal bills showing what would be discussed with his counsel. Also, John may defend this claim by trying to prove that the Insurance Contract only applies to current officers of the Company, therefore he is not required to use Panel Counsel, and fully cooperated under the terms of the Separation Agreement by using counsel of his own per the Indemnification Agreement. Company will not prevail in a breach of contract action against John.

III. Possible improper attorney conduct

The Company’s General Counsel may have acted improperly and could have light sanctions imposed on him by the ethics committee of the State bar.

The issue is whether it was improper to insist on Attorney-Client privileged documents, such as detailed legal bills, from John and his attorney.

The General Counsel approved John’s attorney in writing and agreed to indemnify John as required by the Indemnification Agreement. Therefore, the General Counsel obviously believed that Agreement to be in full force and effect. In addition, later receiving detailed legal bills, which the General Counsel knows are privileged, and refusing to pay such attorney bills, amounts to bad faith and the General Counsel may be sanctioned.

Contracts
Sample Answer #2B

To: Trial Judge

From: Your Law Clerk

Your Honorable, I respectfully submit the following issues for your consideration in this matter.

(1) The first issue concerns whether there was any breach of the 2003 Separation Agreement involving John and Company. A contract is formed by parties who mutually assent to its terms in return for mutual consideration. As a rule, a breach of contract occurs when either party fails to perform under the agreement, or attempts in bad faith to impair or frustrate the terms of the agreement.

In the present matter, a valid contract existed. John was paid a severance in consideration for his agreement to cooperate in future litigation. Both parties assented in writing. The terms of the contract provided that John’s failure to cooperate would result in forfeiture of the severance. 

In 2002, this contract was implicated when litigation ensued against the Company and John. John made good faith attempts to comply with his contractual obligations. He hired an attorney, which Company approved. The Company further agreed to pay John’s legal bills as a result of a 1999 Indemnification Agreement.

However, the Company required John to submit “detailed” legal bills. When John failed to do so, the Company revoked approval of John’s attorney and considered John to be in violation of the contract. These “detailed” bills were required by the Company to contain legal steps and defenses on John’s behalf.

Given these facts, John was not in breach of his contract. He had complied with his obligations to cooperate with the investigation by hiring an attorney. He submitted legal bills as requested. Neither the 1999 agreement nor the 2003 separation agreement required the submission of “detailed” bills as their terms.

However, the Company will argue that John was required to use an Insurance Company attorney from an approved list as required by a 2001 agreement between Company and Insurer. However, John’s contract did make any mention of his obligation to do so; and, moreover, Company waived the right to require John to seek an Insurer-approved attorney when Company approved, in writing, John’s choice of counsel.

In light of these considerations, John likely did not breach the terms of his contract with Company.

Company, however, breached its contract with John by requiring him to provide “detailed” bills. Bills containing legal notes and strategy prepared in anticipation of John’s lawyer are considered protected material under the work-product doctrine. This doctrine absolutely protects any notes, strategy, defenses, opinions or other litigation tactics from disclosure. Thus, neither Company nor its attorneys has any right to obtain them under the guise of requesting a “detailed” bill.

The Company and its attorneys may argue that such product is necessary in its own defense of the claims. However, such an argument fails because the work-product doctrine does not provide for such an exception without the client’s consent; and, John’s interests may be adverse to the Company’s interests in the suit and therefore such disclosure could harm his legal interests.

(2) The second issue. Furthermore, Company’s attorneys’ behavior constitutes an ethical violation. Lawyers are prohibited from communicating with a party represented by counsel.

Here, Company’s counsel contacted John directly and wanted to discuss strategy. Thus, they violated the rules of professional conduct.

In sum, Company breached its contract with John by refusing to pay his legal bills unless he provided privileged information. Thus, I respectfully recommend Your Honor award John $255,000 in legal fees as a result of this breach for his damages. I also request this Honorable Court order company to allow John to keep his present attorney and make no further unethical or unlawful demands upon them.


 

Question #3
Property


Jim owns two adjoining parcels of land in Atlantic Town. The first (“Parcel 1”) is undeveloped beachfront. Jim lives on the second (“Parcel 2”) in one unit of a two-family house built in 1940. Parcel 2 has no beachfront so Jim uses a path across Parcel 1 to access the beach.

Jim decides to sell Parcel 2. Phil has always wanted to live near the beach but knew he needed a two-family house for the rental income. Phil buys Parcel 2 from Jim. Jim tells Phil he may use the path across Parcel 1 to access the beach, as there is no other access nearby.

After selling Parcel 2 to Phil, Jim builds a house on Parcel 1 and sells Parcel 1 to Bob. Wanting more privacy, Bob creates an eight-foot-high sand dune on Parcel 1, which he tops with six-foot-high trees. The dune blocks both Phil’s view of the ocean and the path across Parcel 1 to the beach.

Concerned about the environment and wanting to reduce the amount of rubbish he produces, Bob creates a compost area in his yard. The compost area emits extremely noxious odors. The sea breezes blow these offending odors onto Phil’s property. Phil can no longer use his yard because these odors nauseate him and anyone else who enters the yard.

In order to preserve views of the sea, Atlantic Town has an ordinance prohibiting erection of a fence over four feet.

Phil was recently cited by Atlantic Town’s zoning officer for violating the 1950 zoning ordinance that permits only sing-family homes in the area where Parcel 2 is located.

Phil retains your law firm. He wants to know what his rights are concerning the path across Parcel 1, what he can do about the dune on Parcel 1, and whether the compost area can be eliminated. He also wants to know how to respond to the zoning officer’s citation. You are asked to write a memorandum outlining Phil’s rights and obligations.

Prepare the memorandum

Property
Sample Answer #3A

MEMORANDUM

To: Phil
From: Lawyer
Date: March 1, 2007
Re: Parcel 1 issues

Path across Parcel 1 

Phil has the right to continued use of the path easement either by easement by implication, necessity or grant. The issue is whether an easement existed.

An easement is a possesory interest to go on the land of another for personal uses. An easement appurtanent requires both a dominant and servient tenement. The dominant tenement is the tenement that gets the benefit of the easement and the servient tenement is the tenement that is burdened by the easement. Easements may be created in four ways which include prescription, necessity, implication, and by grant. Easements may be terminated by estoppel, necessity ends, destruction, condemnation, release, abandonment, merger and prescription. An easement by implication is an easement existing by prior use. An easement by implication typically arises when there are two parcels owned by a common owner and they are later divided. The new owner reasonably expects to be able to use a path or other like rights as it has used before the properties were split. An easement by necessity arises typically when land is landlocked or when there is no other access for ingress/egress from the property. An easement by grant arises when the dominant landowner explicitly grants the other person an easement to use the land.

Here the facts state that Jim owned two adjoining parcels of land, Parcel 1 and Parcel 2. Parcel 2 has no beachfront so Jim used a path across Parcel 1 to access the beach. Phil bought Parcel 2 from Jim and Phil told him that he may use the path across Parcel 1, as there is no other access nearby. After Jim sold the house to Bob, Bob constructed dunes that blocked the path across parcel 1 to the beach. Phil will argue that there was an easement by implication, an easement by necessity or an easement by grant. Phil's chances of success would be greatest by implication or grant. Phil reasonably expected and relied upon the right to use the path after the property was sold. It was an apparant use that should be able to be continued. However, Bob will counter by stating that he was not required to continue to allow Phil to use the path. There was nothing explicitly in his deed stating that this easement existed. The facts simply stated that Jim "told" Phil that he may use the access to the beach. Phil will counter that the easement is implied by prior use and Phil had a legitimate expectation of being able to continue to use the path for access to the beach, even after Jim sold the parcel to Bob. It is extremely likely that Phil will be found to have the right to use the path and the court would require Bob to make accomodations because the easement by implication was not terminated.

Phil could also try to make the argument that he recieved an easement by grant. However, Bob will argue, as noted earlier, that there was nothing in his deed stating that an express easement has been granted and that he is not required to follow an oral promise made by an earlier landowner. Typically an easement by grant is recorded to make future purchasers aware. It seems that Phil will have a tough time arguing that an easement by grant existed because there was no writing. The statute of frauds requires that an interest in land (i.e. easements) of more then one year must be in writing. The other argument that Phil could try to make is an easement by necessity because he has "no other access to the beach." Typically, easements by necessity are used for ingress/egress to get to a highway. However, Phil could argue that because he has no other way to get to the beach, he should acquired an easement by necessity. This argument could also be successful because there is no other reasonable access and this is a beach home. By owning a beach home, the person should have access to the beach.

Phil has the right to continue using the path easement over Bob's land by either implication or necessity.

Dune on Parcel 1

Phil would be able to seek relief about the eight-foot high sand dune which he tops six-foot high trees because they are in clear violation of the purpose of the zoning ordinance which requires no fence erection over four feet. He will be able to seek an injunction to remove the dunes/trees and/or get the municipality to enforce their ordinance.

Typically, a person is free to use his land how he sees fit within reason. However, when there is a zoning ordinance requiring action the person cannot knowingly violate it. A municipality, under their police powers have the right to make reasonable restrictions on the use of land.

Here, Atlantic Town has an ordinance prohibiting erection of fences over four feet. Bob, created an a dune that was eight feet high and topped it with trees six feet high. This equals over 10 feet of obstruction when the ordinance requires no fence be over four-feet in order to preserve the views of the sea. Bob will argue that he is not in violation of the zoning ordinance because he does not have a "fence" that is over four feet. However, although he is technically in compliance with the letter of the ordinance, he is in violation of the spirit and purpose of it. His dune and trees are obstructing the views of the sea in violation of the ordinance.

Phil's other argument could be that there was a negative covenant on the land of Bob. There are four recognized negative covenants, light, air, support and stream water from an artificial flow. However, Phil's argument will fail because in order to have a negative covenant, it must be expressly provided for. Here, although the dune and trees obstruct his view, it is not expressly provided for and not clearly recognized to have a view of the beach. This won't be Phils strongest argument.

Phil can either try to seek an injunction to have the dune and trees removed and/or he can try to have the municipality take action to enforce the ordinance. In any event, Bob will be required to remove this obstruction.

Compost Elimination
Phil can successfully seek equitable relief to have Bob remove/halt the use of the compost. The issue is whether the compost is a nuisance.

A nuisance is an unreasonable condition/nuisance on the land that affects a neighbors use and enjoyment of his property. The condition must be unreasonable to a normal person and cannot be claimed when the person has a hypersensitivity. A court will apply a burden-benefit analysis to determine whether the benefit of maintaining the condition on the land outweighs the harm that it causes. A plaintiff claiming a nuisance can seen equitable relief to stop the unreasonable condition that affects the use and enjoyment of another. Equitable relief is appropriate when money damages are inadequate and when enforcement is feasible. A plaintiff also has the right to seek money damages for the damage caused to his land.

Here the facts state that Bob created a compost area in his yard because he wanted to reduce the amount of rubish that he produces. The compost area emits extremely noxious odors and the sea breezes blow these offending odors onto Phil's property. This can be said to be an unreasonable condition on the land that affects Phil's use and enjoyment of his property. The burning of trash in public that creates a noxious odor would be disturbing and affect the use and enjoyment of property by any reasonable person. There is nothing to imply that Phil is hypersensitive to the condition. Therefore, the court will look to see whether the burden outweighs the benefit. The facts do not make clear why someone would need to create a compost rather then just reducing rubbish by having the trashmen pick it up. The harm of having a compost that emits these noxious odors is substantially outweighed by the harm that it is causing to Phil's use and enjoyment of his land. Therefore, Phil may seek equitable relief to get a court to order that Bob cease use of the compost. Equitable relief would be the preferred method here because money damages would not be adequate under the circumstances.

Therefore, Phil can successfully argue that Bob has created a unreasonable nuisance that must be stopped.

Zoning Officer's Citation

Phil should argue that he is not required to conform to the zoning oridance because he has a prior existing use. The issue is whether Phil would have to follow a 1950 zoning ordinance when the house that he bought was built in 1940, 10 years before the ordinance.

A state or municipality under their police powers has the right to make reasonable restrictions on land use. This power is not unlimited but the municipality has broad discretion to make land use restrictions as long as they are reasonable. Most municipalities have master plans that set forth the restrictions and also have zoning boards who make the regulations and hear complaints. A municipality cannot require a person to eliminate a pre-existing non-conforming use all at once if it would be unreasonable. If it is grossly unreasonable, it could be considered a taking that requires just compensation.

Here, Phil bought the land from Jim. Jim had built the two-family house in 1940. The zoning ordinance was not passed until 1950. Therefore, there is a pre-existing non-conforming use on the land. There is nothing that Phil could reasonably do to come into compliance with the zoning ordinance because that would involve the destruction of his home. If the zoning board required him to come into compliance, this would be a taking and Phil would be entitled to just and fair compensation.

Phil should respond to the zoning officer that he does not have have to conform to the ordinance because he has a prior existing reasonable use.

Property
Sample Answer #3B

MEMORANDUM

To: Phil

From: law firm

Re: Phil's rights and obligations

Path across Parcel 1

A covenant arises where a servient land can be burdened for the benefit of the dominant land. If the covenant touches and concerns the land, then it is bound to all successive owners of both land. However, the covenant must be in writing or must be clearly visible to the reasonable person in that land as to require the buyer of that land to make an inquiry of the existence of such path. An easement can be created by necessity where a landlocked parcel has no access to the public road or to the outside. This does not necessarily have to be in writing, however, the easement must be so obvious to allow its existence and use by the dominant land. This easement touches and concerns the land as there is no other way to get to the public highway or the outside world.

In this case, Jim owned the land himself (both Parcel 1 and Parcel 2). Although he used the path across Parcel 1 to get through to the beach from his home in Parcel 2, there is no easement because he is the sole owner of both parcels. There is no dominant and servient land but just one land. When Jim sold parcel 2 to Phil, since Parcel 2 did not have access to the beach and there was no other access nearby, Parcel 2 automatically had an easement out of necessity over Parcel 1 - the path that Jim used to use to get across to the beach. What would be the point of buying a house near the beach but with no access to the beach. The path is there, it is obvious and Jim told him that he may use the path across Parcel 1 to access the beach.

When the parcel 1 and 2 were severed (Parcel 1 became the servient land for the benefit of the Dominant parcel 1) parcel 2 needed access to the beach, the easement arose out of necessity. Therefore, Phil clearly has the right to use the easement - the path across Parcel 1.

Sand dune on Parcel 1
Because Phil has the right to use the path across Parcel 1, Bob cannot unreasonably interfere the use of this path by the dominant parcel. Creating a sand dune of eight-foot high and on top of that trees that are six-foot high is unreasonably interfering with Phil's access to the beach by blocking the path across parcel 1.

In addition to that, it is interfering with the quiet enjoyment of the reasonable use of space that he is entitled to as a property owner of land near the beach. The purpose of buying houses near the beachfront is to enjoy the nice view of the beach. It cannot be unreasonably blocked or interfered with the use of this space. Although, not in every case there would be a right to air space or enjoying the view of your windows (such as in tall apartments in big cities such as NY or Chicago). However, in this situation the unreasonable interference of air space use and blocking one's view of the beach would not be allowed.

There's also an ordinance that prohibits the erection of a fence over four feet. Although the sand dune is not a "fence" the purpose of the ordinance is "to preserve views of the sea." By creating the sand dune, Bob is also violating the ordinance by blocking the view of the sea that Phil is entitled to.

Whether the compost can be eliminated
A nuisance is the unreasonable interference with the use of another's land. The interference must be very unreasonable such as in this case where Bob's use of his own land is created extremely noxious odors as to cause Phil to no longer use his yard because of nauseating effects to him and anyone else who enters the yard. This would be a private nuisance since it is interfering with the use of Phil's private land (unlike a public nuisance where a factory is emitting foul odor and affecting the whole community nearby).

Phil can have a claim again Bob for private nuisance and can either have injunction or damages. However, an injunction would be desirable in this case because we are trying to stop Bob from further creating the extremely noxious odors that unreasonably interferes with Phil's reasonable use of his own land. Therefore, the compost can be eliminated.

Zoning officer's citation
The zoning officer's citation is based on a 1950 zoning ordinance which was enacted after the house was already built in 1940. Phil can argue that the 1950 zoning ordinance doesn't apply in this case because the house was already built and that it would not be feasible or even reasonable to destroy and build another house just to conform to the zoning ordinance. Phil could ask for a variance that would allow him to stay as a two-family house without violating the zoning ordinance. It will be likely that the zoning board would grant this variance. Phil must also know that if the house gets destroyed somehow and a new house has to be built then the zoning ordinance applies and Phil must confirm to it by building a one-family house. A variance will no longer be applicable in this case and he would have to stick to the one-family house as required by the ordinance.

Question #4
Constitutional Law

 

Ashley is an investigative reporter looking into the recent murder of Brian, the husband of Stacey, a famous corporate executive. Ashley interviews John, Stacey’s longtime chauffeur, and learns that John hired a hit-man to kill Brian. In the interview John admits he did it as part of a plan hatched by Stacey for insurance money. In an interview with Mark, the paramedic at the scene, Ashley learns that just moments before his death, as Brian lay on the sidewalk bleeding, he stated, “I don’t think I’m going to make it. My wife Stacey is embezzling millions from her company even while she makes them billions; I was on my way to tell the authorities. Let them know!”

In their investigation, police find Brian’s diary, which details comments Stacey made during their evening nightcaps that led him to believe she was embezzling funds. The police, using Ashley’s interview tapes, convince John to confess his part in the murder. The confession implicates Stacey, who is then charged with Brian’s murder.

At her trial, the prosecution seeks a ruling from the court regarding the admissibility of the following: (1) the testimony of Mark; (2) the confession of John; and (3) those portions of the diary that detail the conversations between Stacey and Brian. Stacey’s attorney objects to all the evidence. The court admits Mark’s testimony but excludes John’s confession and Brian’s diary.

Stacey’s attorney seeks a ruling regarding the admissibility of the following: (1) Mark’s recent conviction for selling drug prescriptions; and (2) testimony from Chief Executive that Stacey heads her company’s extensive charity efforts after Hurricane Katrina and the recent tsunami disasters and is still considered the company’s most valued executive. The court excludes Mark’s prior conviction but admits the Chief Executive’s testimony.

The jury convicts Stacey, who timely files an appeal. You are the appellate judge’s law clerk assigned to write a memorandum analyzing all of the trial court’s evidentiary rulings, citing the factual bases and all applicable rules of law

Prepare the memorandum

Constitutional Law
Sample Answer #4A

In regard to the Prosecution’s sought rulings, the court should find the following:

1) The testimony of Mark is inadmissible under the Hearsay Exception of Dying declaration.

Relevant testimony is any evidence which tends to prove to disprove an element evidence is Admissible as a general rule; However, there are many exceptions including Hearsay. Hearsay is an out of court statement offered to prove the matter asserted. Hearsay is generally inadmissible however there are many exceptions.

A Dying declaration is an out of court statement made by a declarant in the face of impending death. The judge should rule that the test for Brian’s statement was subjectively met. Further Dying Declarations are admissible in criminal trials and the statements must pertain to the circumstances surrounding the death. This exception is allowed because dying people are supposedly less likely to lie in the dace of death.

Brian’s testimony is relevant because it tends to show motive for Stacey wanting to kill him. The testimony that Stacey was embezzling money and he was on his way to tell the authorities. In additional Stacey is on trial for murder not embezzlement so the prosecution can argue that the statement is being introduced to show motive rather than truth of the matter asserted.

In this case because Brian’s testimony is relevant and goes to show both motive as well as possible circumstances surrounding his death, the evidence should be rules admissible.

2) John’s confession should be ruled admissible against him but with limiting instruction, providing his Miranda rights were not violated.

In order for John’s statement to be used against him, it must have been procured in accord with his constitutional rights. The facts do not state otherwise.

Under modern law, a defendant can not be convicted of a crime based solely on uncorroborated testimony of a co-defendant.

In addition for a prior testimonial statement to be admissible as Substantive evidence, the party in which the testimony is introduced against, must have a right to challenge or cross-examine the declarant.

In this case, the prosecution seeks to introduce hearsay to prove that Stacey was part of the murder plan. There is no exception to this hearsay because Stacey did not have an opportunity to challenge his statements. Therefore, the evidence should be ruled as admissible against John but inadmissible against Stacey.

3) Brian’s diary should be ruled as inadmissible hearsay.

Under the business record exception to hearsay. an out of court statement will be allowed if it was recorded or written in a systematic and continuous manner. These requirements discourage false writings made in anticipation of litigation.

Brian’s diary does not fall into this exception and it therefore inadmissible hearsay.

Further, Stacey will argue that the federal marital privilege applies, however the marital privilege only applies to confidential communications which this is not and the priveledge is revoked because she is on trial for murdering her husband.

4) In regard to Stacey sought ruling. 
(1) Mark’s recent conviction for selling drug prescriptions is admissible. A party may impeach a witness with any felony within the past ten years. After 10 years a party may still introduce it but only with permission of the court.

Assuming selling the prescription drugs is a felony and because his conviction is recent, Stacey may introduce evidence of Mark’s recent conviction for Impeachment purposes.

2) Stacey may introduce evidence of her employment but the evidence of her Hurricane Katrina charity is inadmissible hearsay.

Although Evidence to show propensity of a parties conduct based on prior conduct is inadmissible, a defendant may introduce evidence of her good character to show that she acted in comformity with that good character. It should be noted however that once defendant raises character and introduces evidence as such, the opposing party may rebut that evidence by showing defendant bad character; Defendant “opens the door.” However, a party may not raise specific acts as means of showing character evidence of this character must be introduced as reputation or opinion under the Federal Rules of Evidence.

In this case Stacey has offered to show her good character by means of the Chief Executive’s testimony through the specific act of her charitable volunteering on Hurricane Katrina and Tsunami projects. This should be viewed as hearsay as well as a specific act and should therefore not be allowed. Further her charitable contributions are irrelevant as to the charge of Murder, a violent crime.

The Statement from the chief executive that Stacey’s reputation and his opinion of her was esteemed is admissible. First this testimony is admissible as good character evidence and second as relevant testimony.

Stacey’s reputation of being a highly valued employee goes to the underlying motive that the prosecution is seeking to raise. Stacey’s good work ethic is relevant to the charge of embezzlement the Company’s money. Further it shows Stacey’s good character. It should be noted however that once Stacey “opens the door.” The prosecution may attack her character with bad character evidence.

Constitutional Law
Sample Answer #4B

MEMORANDUM

To: Appellate Judge
From: Applicant
Date: March 1, 2007
RE: State of New Jersey v. Stacey; Evidentiary Rulings

Prosecutorial Motions to Admit Evidence

1) The state seeks to admit the statement of Mark, a paramedic at the scene who had a conversation with the decedent moments before he died. The statement implicated the decedent’s wife in an embezzlement scheme, and potentially goes to motive of the crime.

The issue is whether the statement made by Mark is hearsay, and if it is, whether it should be admitted under the “dying declaration” doctrine. Hearsay is an out-of-court statement, offered by someone other than the declarant that is offered at trial to prove the truth of the matter asserted. The general rule is that hearsay is not admissible unless it meets an appropriate exception. In this case, Mark is testifying to an out of court statement, made by Brian, and is offering it to prove the motive of Stacey’s alleged murder of John, which is an admissible use of the statement. If, however, the statement is being offered as substantive evidence that Stacey embezzled from her company, it will not be admissible. The judge should instruct the jury with an appropriate limiting instruction, because the statement that Stacey embezzled lacks reliability.

A dying declaration is admissible for substantive purposes in a criminal homicide trial. The presumption is that a person has no incentive or motive to lie moments before their death. However, the statement must go directly to the cause of the decedent’s death, and the declarant must subsequently die.

Here, while Brian did pass away moments after making the statements, it is questionable whether his statements identify his killer. Had Brian said “Stacey and John killed me,” then the statement would be admissible for substantive purposes against Stacey. However, the ambiguity John’s comments allow the court discretion in deciding whether or not to admit the entire statement, and the statement probably does not qualify under the dying declaration exception. It may nonetheless be admissible only to establish motive.

2) The state wishes to admit evidence of John’s confession, which implicates Stacey. Assuming no 5th or 6th Amendment violations in obtaining the voluntary confession, the confession of a co-conspirator is admissible against his co-conspirator if the confessing party is available to testify.

The issue is whether Stacey’s Sixth Amendment right to confront her accuser will be satisfied. The rule is that a criminal defendant has a constitutional right to cross examine her accuser if the proceeding is “testimonial.” A criminal homicide trial is “testimonial.”

In this case, John is available as a witness, and may be cross-examined and impeached on the basis of his bias. Stacey’s attorney has the right to ask John about any incentives that he has to lie during his testimony. This will satisfy Stacey’s right to confront John, and therefore, John’s confession is admissible for substantive purposes against Stacey.

3) The state wishes to admit into evidence portions of a diary that detail conversations between Brian and Stacey that tends to implicate Stacey in embezzlement. The issue is whether the diary fits the hearsay exception of past recollection recorded. The rule is that in order for hearsay (definition above) to be admitted as a past recollection record, the declarant must be unavailable, and the recorded recollection must be reliable.

In this case, Brian’s diary was a privately kept memoir, in which he would have no reason to lie. Brian is unavailable as he is dead, and so the diary would be admissible under prior recollection recorded, unless the information contained therein is privileged.
The issue is whether the substance of Brian’s diary is privileged communication between husband and wife. The rule is that, in a criminal proceeding, any confidential communications made between married couples is barred from admission, unless both spouses waive the privilege.

In this case, the contents of the diary are recorded conversations that took place during “evening nightcaps” between Brian and Stacey, married parties. Because the conversations were private and confidential, Stacey holds a valid privilege in the diary’s substance. The contents of the diary are not admissible.

The Defendant’s Motions

1) The defense wishes to admit evidence of Mark’s recent conviction for selling drug prescriptions. The issue is whether Mark’s conviction was a felony or a misdemeanor. The rule is that a defendant has a right to impeach any witness at a criminal trial for any felony conviction that was entered into judgment within the last 10 years. Most jurisdictions will also permit misdemeanor convictions that call into question the truthfulness of the witness that occurred within the same period. The judge in the case will also have to determine that the probative value of the evidence is not substantially outweighed by the prejudice caused to the witness before admitting evidence of the conviction. Extrinsic evidence in a criminal trial is admissible to prove the conviction if the defendant denies the same.

In this case, Mark was convicted of selling drug prescriptions in his capacity as a paramedic. If the conviction was a felony (one punishment by imprisonment of more than a year), then in the State of New Jersey, the conviction is per se relevant, and is always admissible against the witness. If the crime was a misdemeanor, then the judge must perform the balancing test. In this case, the evidence of the conviction is probative, because it impeaches the credibility of the witness, but is not overly prejudicial that the jury would disregard at hand anything Mark has to say. Therefore, Mark’s conviction is admissible.

2) The defense seeks to suppress statements made by Chief Executive about Stacey’s character and opinion. The rule is that character evidence of specific acts is never permitted on direct examination to prove that the person in question acted in conformity that that character trait. In this case, Chief Executive’s statement that Stacey heads the charity division of her firm is inadmissible because it refers to a specific act on direct examination, and is offered to prove conformity.

The second part of Chief Executive’s statement that Stacey has a reputation as the “most valued executive” calls into question a separate issue. Evidence of the defendant’s reputation in the community is admissible on direct, because it is probative to the jury. However, once Stacey “opens the door,” to her reputation in the community, the state will then be able to admit reputation or opinion evidence to impeach that same reputation. The second part of Chief Executive’s statement is admissible.

Question #5 
Civil


The New Jersey Economic Development Authority (“Authority”) solicited bids for the construction of a state government building (the “Project”), which state law requires be awarded by a sealed bidding process, to foster competition. The procedure requires all bidding contractors to submit sealed bids on the bid date, which must name all subcontractors they intend to use. The contract is awarded to the lowest responsible bidder after all bids are opened and reviewed. The Project went to bid with the stipulation that final award of the contract was contingent on appropriation by the Legislature and of the necessary funds, which approval was pending but not certain at the time of the bids. After all bids were opened and reviewed, Low-Bid Contractors was identified as the lowest bidder, to be awarded the contract if and when the monies were appropriated. Several weeks after the bids were opened, Low-Bid gave notice that it was changing certain of the subcontractors it had named in its bid proposal. The Authority permitted the substitution under an internal policy it claimed gave it discretion to allow such substitutions.

Ace Contracting was the second-lowest bidder. Best contracting did not bid the Project but typically does bid similar state construction projects and has in the past performed such contracts. NJ Contractors Association (“Association”) is a trade association of contractors who perform similar work, to which Association both Ace and Best belong.

Ace, Best, and the Association filed suit in federal district court seeking, among other things, (1) to enjoin the Authority from awarding the contract to Low-Bid, on the basis that substitution of subcontractors after the bidding constituted an impermissible change of the bid; and (2) to declare the Authority’s policy allowing substitutions void as violating the public bid laws.

Part 1

You are a junior attorney representing the Authority. You are assigned to identify all procedural defenses the Authority can raise against the above two claims, as to each of the three plaintiffs, and to discuss their elements and their likelihood of success as to each plaintiff separately. You are to assume for purposes of this assignment that the complaint presents a federal question. You should not address federal question or diversity issues in your memorandum, but address any other jurisdictional issues.

Part 2

Shortly after you complete the memorandum, the appropriations bill fails, and the contract therefore will not be awarded. Your supervising attorney directs you to prepare a supplemental memorandum addressing the impact this development has on the case, if any.

Prepare the memorandum and supplemental memorandum

Civil
Sample Answer #5A

MEMORANDUM

Claim 1 – Enjoin Authority from Awarding the Contract to Low-Bid
Governmental Immunity
We may raise a defense of governmental immunity. A citizen may not sue the government for the conducting of its duties without a waiver of immunity. This defense protects government actions within their discretion. In this case, the plain language of the law does not ban a stipulation. Rather the law mandates a procedure for the sealed bidding process. The Authority, in its discretion, has created an internal policy that allows for its use of discretion in allowing substitutions of subcontractors named in a bid proposal. Because the Authority acted within its discretion, properly outlined in its procedures, we should successfully raise a governmental immunity claim against all three parties.

Lack of Standing 
We should also raise the defense of the parties lack of standing. To have standing a party must show injury, causation, and redressability. Third party-standing is generally not allowed. First, a party must show that they were injured in fact. Second, they must show that the challenged act was the cause of their injury. Finally, they must show that judicial action may redress their injury.

This issue is separate as to each plaintiff. Because they were the second lowest bidder, Ace Contracting may succeed in establishing standing. Conversely, Best Contracting did not bid on the Project. Even though they typically do bid on similar projects, they have no injury in fact. Thus, there is no causation nor redresssability as to this issue. Finally, the Association may not have standing. There is an exception which allows for third party standing for associations. The association must prove that its members were harmed by the act in question and those members are not raising the claim on their own behalf. Here, there is no pleading that any of the contractors in the Association, besides Ace and Best, are in the Association. Thus, a court may rule that the Association has no standing.

Failure to State a Claim
Finally, as to the first cause of action, we should raise the defense of failure to state a claim. In this case the plaintiff’s legal theory is based on a theory that it is impermissible to change the bid after the submission of the sealed bid. The plain language of the statute does not support this assertion. The statute merely the procedure for the sealed bidding process. It is silent on the post-submission changing of bids. Thus, there is no actual violation of the statute. Consequently, each of the Plaintiffs have failed to state a claim upon which relief may be granted.

Claim 2 – Declare the Authority’s Policy Allowing Substitutions Void
Lack of Standing
Normally, citizens do not have standing to challenge a government act unless they are harmed by that act. In this case, only Ace may potentially be harmed by the policy. Thus, both the Association and Best likely do not have standing as to this claim.

Failure to State a Claim
Generally, the legislative branch of the government has a broad power to delegate authority to

SUPPLEMENTAL MEMORANDUM
The failure of the appropriation bill makes the first claim moot. Because there no longer is a contract to award, there no longer is any valid claim for enjoining the Authority from awarding that contract. Thus, as a matter of law, this issue should be dismissed.

Civil
Sample Answer #5B


Part 1

TO: Supervising Attorney
FROM: Junior Attorney
Memorandum re

Matter of NJ Economic Develop. Auth.

In general in order for a federal court to exercise its judicial power over a claim there has to be standing of the plaintiff so the federal court may hear the case; ripeness; and the issue should not be moot.

In order to have standing a plaintiff has to show that he has suffered an injury or he is about to suffer injury. The injury does not have to be a physical injury and economic damages are sufficient to establish standing.

As to ripeness, the issue before the court has to sufficiently developed.

As to mootness, the issue before the court has to be still a valid issued requiring a resolution unless it falls into one of the exceptions, such a conduct capable of repetition or voluntary discontinuance.

In this case, three different plaintiffs are suing the defendant (Authority).

As to Ace (A), there is standing because A may show that he suffered economic harm because of Authority’s conduct. If Authority could not allow the change in sub-contractors after a bid is won, then Low-Bid may not have won the contract with Authority. Indeed, Low-Bid’s changes to the subcontractors were made only after the award, and if the changes were made before there was a chance that Low-Bid would not get the contract, and consequently, A as a second lowest bidder would get it instead.

The issue is ripe. The internal policy that is being challenged has been used to the detriment of A, and therefore it is not premature for the court to address the merits and the validity of the policy.

The issue is not moot because the challenged policy is still in place and raises questions that have to be addressed by the court. Also A’s case does not fall into any of the exceptions for mootness.
Accordingly A’s case will probably be heard by the federal court, and the Authority is unlikely to successfully challenge it on procedural bases.

As to B’s claim, B lacks standing to ask for injunction and to declare the policy unconstitutional. Because B did not bid on the contract he was not adversely affected by the policy or by the way it was enforced. B’s claim is ripe, and not moot, but without standing, the court would not even have to look at ripeness and mootness. Accordingly, B is likely to succeed on its challenge to the court’s jurisdiction over B’s claim. (Please note that the ripeness and non-mootness are identical for all three defendants).

As to the Association, its claim against Authority is ripe and non-moot as per discussion above. The issue is whether an organization has standing to sue in federal court on behalf of its members, if individual members have suffered an injury. Generally the answer is yes, and Association would have standing to ask for injunction of the contract award to Low-Bid and to challenge the validity of the Authority’s policy, because its (Association’s) members, such as A, were injured as a result of the policy application. The injury results from award of contract to a lowest bidder who can later change details of his bid, thus prejudicing other bidders who only have a single opportunity to formulate the bid. Accordingly, Authority is not likely to prevail in its attempt to have the Association’s claim dismissed based on procedural grounds.

Part 2
Supplemental Memorandum
In view of recent developments, in particular that the appropriation bill has failed, the matter discussed in the above memorandum has to be reviewed. Authority may challenge the actions of A, B, and Association on the ground of mootness, because since there is no funding for the project, no one was awarded the contract (as it was an express condition of the award) and therefore the issue is moot. However, even though the issue is moot as to the injunction, it is not moot with respect to the review of the policy. The application of the policy the Authority is conduct capable of repetition, because Authority may act in the same way pursuant to the policy on the next contract and request for bids. Therefore, the issue is not moot for claims of A and Association with respect to challenging the validity of the policy. Accordingly, Authority is likely to succeed in dismissing only the claims seeking injunction, but not the claims challenging the policy on the ground of mootness.

Question #6
Criminal


Steve and Matt are identical twins who attend Bell High School. Matt is a serious student. Steve is a wrestler, practical joker, and bully. Jim is a student who wears thick glasses. Betty is tall and significantly overweight. Steve has taunted, hit, and bullied Jim and Betty for many years.

Steve saw Betty by herself in the school corridor. Brandishing his pocket knife, Steve told Betty he could help her lose 20 pounds real fast. Steve lunged towards Betty, who recoiled, tripped, and hit her face on the corner of the trophy case. Steve ran towards the boys’ room.

Betty was holding her nose, which was bleeding profusely, as Jim approached. When Betty told Jim what Steve had done, Jim got very upset. He said, “Enough. Steve can’t break your nose and get away with it. I’m gonna kill him.” Betty replied, “Don’t – he’ll hurt you too.”

Ignoring Betty, Jim stormed off to find Steve. As Jim approached the restroom, he started to think Betty was right. He ducked into a doorway to think things over. Jim heard a door slam nearby and a deep voice saying, “What do you think you’re doing?” Thinking the voice belonged to Steve, Jim started to run away towards the double glass doors of the gymnasium.

When Jim approached the glass doors, he thought he saw Steve on the other side. Unbeknownst to Jim, it was Matt, not Steve. Rather than stopping, Jim crashed through the doors, knocking Matt to the gymnasium floor. Shortly thereafter, Betty arrived.

Thinking Matt was Steve, Jim said, “We have to do something.” He asked Betty to open the equipment closet adjacent to the glass entry and find some rope, which she did. Jim pulled Matt inside and tied his hands behind his back. Matt started to moan loudly. Still thinking Matt was Steve, Jim told Betty to “get something for Steve’s mouth.” She returned with a wrestling singlet, which Jim stuffed into Matt’s mouth. Jim and Betty then ran away.

Matt continued grunting and gasping. Finally, two wrestlers found him. They pulled the singlet out of Matt’s mouth and yelled for help. The student trainer, Tom, ran towards them to administer first aid. He told one boy to call 911, while the other helped Tom roll Matt onto his back. When Tom saw Matt’s face, he too thought Matt was Steve, and pulled away. He said, “Forget about calling 911. It’s just Steve screwing around again.”

Vice Principal Grant entered the gymnasium to report Steve to his coach for smoking in the boys’ room. Seeing the commotion, he asked, “What’s wrong with Matt?” Realizing Matt, not Steve, was on the floor, Tom immediately tried to help, but it was too late. Matt died from an allergic reaction to the latex in the singlet.

The Bell County Prosecutor has requested a legal memorandum that (a) identifies each person who can be charged criminally as a result of this event, the elements of each criminal offense, and the facts supporting the charges against each; and (b) identifies the elements of each defense and the facts supporting any defenses to the charge.

Prepare the memorandum

Criminal
Sample Answer #6A

Memo to Prosecutor
From Assistant Prosecutor
Re: Persons to be charged, charges, defenses

Steve

Steve can be charged with Assault and Battery. The issue is whether he has any defenses.

Assault is a threat to commit a battery; it is a specific intent crime.

Steven brandished a pocket knife, threatening Betty. He lunged towards her with the knife in hand. His actions constitute assault.

Steve can argue that he is just a practical joker and he was only joking with Betty. He can argue that he did not have the specific intent required to commit the assault.

However, this intent can be shown by the desire for the result or actions undertaken with substantial certainty the result will occur. Lunging at Betty with a knife would be viewed by the victim as an assault. She did not know he was joking and recoiled in fear.

Battery is the harmful or offensive touching of another. It is a general intent crime.

Steve lunged at Betty causing her to recoil, trip and hit her face on the trophy case. He has committed battery.

Steve can argue that he never actually touched Betty and that her injury was caused by her own fall.

However, Steve’s conduct caused the fall and so he is guilty of battery.

Jim

Jim can be charged with Battery, Conspiracy, Kidnapping, False Imprisonment, and Murder.

Battery, as noted above, is harmful or offensive touching.

Jim crashed into the gym doors knowing that someone was on the other side. The doors struck Matt knocking him down. Jim committed battery.

Jim can argue that he did not intend to hit Matt; he intended to hit Steve. But, his intent will be transferred to the victim actually injured. Jim hit the door knowing it would hit Matt. His mistake as to Matt’s identity is not a defense.

Conspiracy requires intent, agreement, and intent to achieve the unlawful purpose of the agreement.

Jim asked Betty to “do something,” and specifically to get rope. His solicitation merged into conspiracy when Betty agreed by getting the rope. Their conduct together shows their intent and agreement. Tying Matt up was the overt act.

Jim can argue that he was acting in self-defense. He reasonably believed that Matt was Steve because they are identical twins. Jim can argue that he did not have an unlawful purpose but only wanted to protect himself and Betty from more harm.

Kidnapping is false imprisonment but involves moving victim.

False Imprisonment is an act of restraint containing an individual to a bounded area.

Jim tied Matt’s hands and then put Matt in the equipment closet. His actions constitute False Imprisonment. Because he moved Matt, it is kidnapping.

Jim can argue that he was only protecting himself and acted out of self-defense because he thought Matt was Steve. However, hiding Matt in the equipment closet exceeds reasonableness and so is not a defense.

Murder is the unlawful killing of another with malice aforethought. Malice can be shown by (1) intent to kill, (2) intent to do serious bodily harm, (3) depraved heart, (4) felony-murder.

Jim is guilty of murder because Matt died during the commission of a felony, kidnapping.

Jim can argue that he did not have intent to kill Matt. However, the intent to commit the underlying felony suffices for the intent for murder.

Betty

Betty can be charged with Conspiracy, False Imprisonment/Kidnapping, and Murder.

Conspiracy was committed when Betty “agreed” to help Jim tie up Matt. Her agreement arose from her conduct. She got the rope. She also got the wrestling singlet to secure Matt’s mouth so he would not scream. Her actions constitute conspiracy.

Betty can argue that she acted in self-defense because she thought Matt was Steve. Steve had just assaulted her and she needed to protect herself. However, her actions are not reasonable and come after there is any threat. They look more like revenge than self-defense.

False Imprisonment/Kidnapping occurred when Betty aided Jim in tying up and then hiding Matt in the closet. As an accomplice, she is liable for the committed crimes, just as the principle. She aided Jim with the intent of helping him restrain and then hide away Matt.

Betty can argue that she acted out of fear and so her actions are self-defense.

However, Matt posed no imminent threat and so her defense is weak.

Murder – Betty will be liable as an accomplice and co-conspirator to all crimes that were foreseeable.

Betty can argue that Matt’s death was not foreseeable because he died from the allergic reaction to the latex in the singlet.

Betty’s argument will be a good defense.

Tom

Tom can be charged with criminal negligence, and involuntary manslaughter.

Although there is no general duty to rescue, once undertaken it must proceed and the defendant cannot abandon the rescue.

Tom began to help Matt. Then, when he thought it was Steve, he stopped. His failure to act is criminal negligence.

Deaths that arise from criminal negligence are involuntary manslaughter. Matt died because Tom did not continue the rescue.

Tom can argue that he mistakenly believed Matt was Steve. But, this mistake is no defense. He can argue that the death was not foreseeable but it may have been if Tom had acted as he should have as a rescuer. He is liable for criminal negligence and involuntary manslaughter.

Criminal
Sample Answer #6B

To: Lead Prosecutor
From: ADA Applicant
RE: Steve case
Date: March 1, 2007

Matt recently died after a string of unfortunate events. Many people have committed crimes, and I will outline the potential liability of each person.

Steve

Steve is Matt’s identical twin. He can be charged with aggravated assault against Betty. Assault is placing a person in reasonable apprehension of an offensive touching or battery. Since Steve did not actually touch Betty, battery, an offensive touching, may not applicable in this situation. The assault is aggravated because Steve used a weapon, the knife, in attempting to frighten Betty.

While battery will be the less likely charge successful against Steve, it may be able to work since Betty was in fact injured by attempting to avoid Steve’s lunge with the knife. Betty broke her nose and had significant harm because of Steve’s aggravated assault.

Jim

Attempted Murder
The crime of attempt is specific intent crime, meaning the person must have the intent to perform a crime. Attempt as a crime merges with the completed crime, meaning a person cannot be convicted of attempt and the crime attempted, such as murder. Murder is the unlawful killing of another with malicious aforethought. Attempt is not merely preparation, but requires a substantial step in the completion of the crime. While Jim went looking for Steve, it does not appear that he made a significant enough of a step. He stopped looking for Steve prior to being anywhere near Steve, he also didn’t get any weapon or instrumentality of the crime. 

If it is found that Jim actually attempted to kill Steve, he would have two defenses. First, defense of another/self-defense. His defense of self-defense would be imperfect because the harm of Jim was not imminent even though Jim had been sufficiently bullied. His defense of defense of another is also imperfect since more harm to Betty was not imminent since Steve had already left the scene of the incident.

Self-defense is only permitted to use reasonable force in response to the crime, and forbids excessive force.

Another defense is that he was not acting with malice, but rather was going to attempt volunary manslaughter. Voluntary manslaughter is the unlawful killing of another without malice, but in reaction to a sufficiently shocking event without time to cool down. In this situtation, Jim did cool down and in the end did not attempt to kill Steve.

Battery
Battery is the offensive touching of another. Jim could have committed battery on Matt when he hit mat with the gymnasium door. Since this seemed to be a surprise and Matt likely had no apprehension prior to be hit with the door, assault does not seem relevant.

Kidnapping
Kidnapping is the unlawful moving of another human being by threat or duress, thereby eliminating the movant’s free will. In this situation, Jim knocked Matt unconscious. When Matt was unconscious, Jim and Betty tied Matt up and gagged him. They then moved him into the closet. This is considered kidnapping since they moved Matt against his will into the closet while tied up and gagged.

False Imprisonment
False imprisonment is the unlawful detainment of another human being without leaving reasonable means of escape. While the door to the closet may have been open, Matt had no means of escape from the closet since he was tied up and gagged. He was unable to move freely or able to call for help. False imprisonment is a specific intent crime. Jim and Betty had the specific intent to place Matt in the closet bound and gagged.

Jim could claim citizen arrest which allows a citizen to detain another for a reasonable period of time after knowledge that the person whom they are arresting actually performed a crime. Citizen arrest likley will not apply since they used excessive force by tying Matt up. In determining the citizen arrest, Jim could claim a mistake of fact since he mistakenly believed that Matt was Steve.

Conspiracy to Kidnap and Falsely Imprison - Betty
Conspiracy is the criminal agreement between two or more persons to commit a crime. In this instance, it could be argued that Betty and Jim were in a conspiracy to falsely imprison Matt and to kidnap him. Each member of a conspiracy is liable for the principle crime.

It could be argued that there was no conspiracy, rather, Betty was an accomplice to the crime. Betty attempted to discourage Jim from harming Steve, but in the end when the situtation presented itself, she assisted Jim in binding and gagging Matt. Either way, Betty may be liable for the crimes performed.

Felony Murder
Felony murder is typically a first degree murder crime – having maximum sentencing ability for the commission of the crime. This is when a victim dies as the result of a violent felony. Violent felonies include rape, kidnapping, aggravated assault, robbery, to name a few. Since Matt was kidnapped and died as a result of the kidnapp because he was gagged with the singlet made of latex which he was alergic to, Betty and Jim may be found guilty of felony murder.

Involuntary Manslaughter
Involuntary manslaughter occurs when the death arises from a misdemeanor offense of an uninumerated felony as listed in felony murder. In this instance, false imprisonment is not an innumerated felony or may be considered a misdemeanor, depending on this jurisdiction (Felony imposes sanction of more than one year imprisonment, misdemeanor is less than one year or no imprisonment). If the jury finds that Betty and Jim did not kidnapp Matt, but merely falsely imprisoned him, they could be found guilty of involuntary manslaughter.

Betty

Conspiracy and Accomplice Liability
Betty may be found guilty of conspiracy if the jury determines that she ultimately agreed to help Jim in the commission of the crime of kidnapping and false imprisonment. If not she could also be found to have accomplice liability since she assisted Jim in the commission of the crime. She can be held liable for the death of Matt since her actions assisting Jim led to Matt’s death.

Betty may have an insanity defense since she was placed in fear of her life by Steve. Under New Hampshire test, any mental defect that could rationalize the crime may be used as a defense. This is likely the only insanity defense that Betty has since she could claim she was traumatized. The Model Penal Code and the M’Naughten test do not apply since she does not have a mental defect with uncontrollable impulses and it appears that she did not have a defect that prevented her from the knowing the difference between right and wrong.

Tom

Cease Rescue
Tom came to Matt’s aid, but upon thinking it was Steve, he ceased the rescue and told others to stop as well. While most states do not have a Good Samaritan law, requiring a person to come to the aid of another in need of help, it is typically illegal to begin rescue but then cease and ordering others to stop, thereby leaving the victim in a worse position than what he would have been otherwise.

In this situation, Tom believed that Steve was playing a practical joke since he knew Steve and mistakened Matt for Steve. This history of practical jokes caused Tom to have a mistake of facts in determining whether to continue the rescue of Matt. Mistake of fact, requires a reasonable mistake of the facts presented.

Question #7 
Evidence



The State of Euphoria recently passed legislation requiring large pharmacies (gross revenues of at least $1,000,000 per year) to dispense “morning-after” contraceptive pills on receipt of a valid prescription. Pharmacies that do not meet the revenue threshold are exempt from this requirement.

Yvette, a licensed pharmacist, works for Titi Drugs, Inc. (“Titi”), a large pharmacy in Euphoria. She refuses to fill several “morning-after” prescriptions because of her religious beliefs, and Titi terminates her employment.

Yvette’s sixteen-year-old daughter, Morena, suffers from a potentially life-threatening medical condition if untreated. However, Yvette objects to Morena’s submitting to traditional medical treatment, including the operation necessary to save her life, based on both her religious beliefs and her preference for a holistic medicinal approach. Euphoria’s Youth and Family Services Division has filed suit to compel Yvette to consent to Morena’s undergoing the life-saving operation.

Yvette is also the pastor in a local church. As an integral part of its services, the church routinely serves wine to Morena and other minors. Euphoria has a statute prohibiting the service of alcoholic beverages to minors. However, this statute is almost never enforced against religious organizations.

Euphoria’s Alcohol Beverage Control (“ABC”) agency wants to make an example of Yvette because of her religious ardor. ABC serves Yvette with an administrative summons for violating the statute. The summons carries a potential $5,000 fine and forfeiture of her church’s tax-exempt status.

Yvette comes to your office seeking legal advice and asks you to prepare a memorandum analyzing her constitutional rights and defenses with reference to all of the foregoing issues.

Prepare the memorandum

Evidence
Sample Answer #7A

Memorandum

Date: March 1, 2007
To: Yvette; client
From: Law Firm

You have presented our firm with several constitutional rights and possible defenses you may have. Below you will find our analysis of those issues as we see fit:

Regarding termination from Titi Drugs—

The First Amendment of the U.S. Constitution protects, among other things, the freedom of religion; (to practice, to believe, to worship, etc.)

However, the First Amendment only applies to the government. Government action is needed before the protections afforded by the First Amendment are triggered.

In our case, Titi is a private employer not subject to the 1st Amend. In our opinion, if your beliefs keep you from dispensing the morning after pill, you may want to seek employment with a small neighborhood pharmacy who does not have to fill such prescriptions. (Note that the regulation requiring pharmacies to dispense the morning after pill only applies to large companies grossing at least $1,000,000 of revenue each year.

Thus, you do not have a constitutional claim against Titi.

Regarding Treatment for your 16 year old daughter:

Under the Constitution, one has a fundamental right to deal with their health concerns, including refusing life saving treatment (although euthanasia is not permitted.)

This, however, is if an adult were making the decision. Although you have an interest and right to practice your religion, the gov’t has a compelling interest in preserving (sic) life – especially when a minor is concerned.

The State has the right to do what is in the best interest of your child in this case. Thus, a court may side with Euphoria’s Youth and Family Services Division in allowing the hospital to proceed with life saving treatment of your daughter.

In your defense, you may argue that you have a fundamental right to raise your child as you see fit. The court will have to balance the interests presented.

Regarding Summons for Violating Service of Alcoholic Beverage to Minors Statute:

If a law is neutral on its face (meaning no apparent constitutional violation reading the law as written) then we have the burden to show that the law has a discriminatory impact and discriminatory effect.

If we are able to show those two things, the State will have the burden of show (sic) that it is not intended to be discriminatory and meet whatever test (either intermediate scrutiny or strict scrutiny) as applies to the violation/discrimination alleged.

In this case, the prohibiting alcoholic service to children is neutral on its face, and its impact and effect are neutral as well. (It is enforced the same.) All the State needs is a rational basis to support the law. (Note that a similar case dealing with Indian pyotie (sic) used in religious ceremonies was challenged and denied.)

However, we may have a case under Equal Protection. The fact that the statute is almost never enforced against religious organization, but is not equally being enforced against you “to make an example out of you because of your religious ardor” is problematic.

Under the equal protection clause you have the right to be treated the same way as others under a given law – and if this one is not being enforced against others – only you, we have a good case against the alcoholic beverage control.

Please advise on how you will like to proceed on the matter.

Evidence
Sample Answer #7B

Memorandum
To: Yvette Client
Subject: Rights re: recent issues

These facts raise several major constitutional law issues.

First, may a state require pharmacies of a certain size to follow certain rules while exempting others of smaller size? The answer is yes. The equal protection clause of the 14th Amendment does not bar states from making legal classification that discriminate on the basis of wealth, whether they apply to individuals or organizations. The classification that distinguishes large pharmacies from small ones need only meet the rational basis test, and here it surely does: large pharmacies inherently have a larger reach, and if the state aims to enforce a public policy, it can solicit the aid of private actors with such reach by limiting their activities adding requirements.

More importantly for Yvette, since Titi Drugs, can therefore be required to dispense “morning-after pills, is it allowed to terminate her employment simply because she personally refuses, on the basis of religion, to participate in the pills’ dispensing? This is a question of interpreting the Establishment Clause of the 1st Amendment, as incorporated by the due process clause of the 14th Amendment. First, the Establishment Clause inherently concerns the government’s actions establishing religious preferences. The only way to prohibit Titi from firing Yvette on constitutional grounds would be to first find that doing so would constitute state action. The governing principle holds that the state cannot force one of its employees to choose between religion and their income. However, to find state action requires that the employer be either a public agency, an organization with significant funding, or an entity whose activities are in some other “entangled” with the state. Here, Titi is a private pharmacy not receiving any funding from the state. Furthermore, the state rule requiring dispensation of the pills does not constitute entanglement with the state, though Yvette would inevitably raise this as a defense. Entanglement is mostly found in the context of schools or avowedly religious institutions. Here, sadly for her, the pharmacy can fire Yvette, because she can always get a similar job at another pharmacy that will allow her to abstain from dispensing these pills. Or she can work for a smaller pharmacy.

The due process clause of the 5th Amendment, as incorporate by the 14th Amendment, also guarantees certain fundamental rights under the rubric of “substantive due process.” One of these is the right of parents to raise their own children in the manner of their choosing. This doctrine would support Yvette’s right to shield Morena from certain live-saving medical techniques if she so chooses, provided she does so based on religious conviction. However, the state validly has an interest in the health and welfare of its children-citizens, and can act on their behalf if parents are placing them in danger. The question is, then, can religious belief justify what would otherwise potentially constitute criminal neglect of a child under the laws of a state? Although this issue has not been specifically resolved by a conclusive federal Supreme Court opinion, the general rule is that states may remove any child from a family pursuant to child abuse and neglect laws, and that religious belief cannot trump those laws. Support for this position comes from the Supreme Court’s Smith case, which holds that the right to free exercise of one’s religion (as guaranteed by the First Amendment as incorporated by the 14th Amendment’s due process clause) is trumped by the state’s interest in fully enforcing laws of general application that are neutral on the question of religion (i.e. laws that neither favor nor disfavor religion in general or any religion in particular). Since Smith places neutral state laws above free exercise rights, by inference, then, child abuse and neglect laws trump the rights of religious people to deny their children life-saving care. Here, Yvette again must yield. She may try to defend herself by arguing interference with free exercise, but again, this is likely to fail.

A similar principle applies to the issue of whether wine may be served to minors in Yvette’s church. Although the free exercise of religion would dictate that the church could serve wine to minors, the Smith case holds otherwise if a state has barred the activity. In Smith, an Oregon statute banning peyote use was held applicable to Native American ceremonies that have used peyote for hundreds of years. Similarly, here, the tradition that Yvette’s church has adhered to must yield to the neutral law of general application prohibiting the service of alcoholic beverage to minors. They may attempt to avoid the Smith logic by asserting a violation of free exercise, but again, they will likely fail.

On the specific question of the church’s penalty, however, the state may have overreached. It is well established law that Congress and the States have acted well within their rights to exempt all religious institutions from a variety of forms of taxation. Although the penalty of a fine seems narrowly tailored to the crime charged – serving wine to minors – denial of tax-exempt status might violate a federal law that preempts state law. Furthermore, violating this one state statute does not, in reality, change the fact that the church is a religious institution. To exempt such an institution would typically require a finding of racketeering and a precise definition of what a “religious institution” actually is, under the law (by statute or court holding). Since no such definition has been attempted in a broad way, many institutions across America are deemed legitimately religious and receive tax-free status, so as to avoid any interference with the Establishment Clause. By invoking such a heavy penalty against the church, the state would probably be implying that the church was itself an improper religion, thus undermining the neutrality at the core of the Smith holding—the holding, which, itself, gives the state the authority to impose this law on the church in the first place. The state may defend itself by calling the penalty a reasonable and neutral reaction, but again, because of its tinge of interference with the Establishment Clause, it will likely fail to successfully remove tax-exempt status from the church.

 



 

Return to Top